Un problème d'analyse intéressant

Bonjour
Je continue mon étude infatigable pour très bien apprendre l'analyse et là j'ai trouvé ce problème.

Soit $f,g\in C([0;1];\mathbf{R})$ et $$\int_{[0;1]}f(t){\rm d}t=\int_{[0;1]}g(t){\rm d}t=1.
$$ Soit $n\in \mathbf{N}$ ,prouvez qu'il existe $a,b\in [0;1],$ avec $a<b$, tel que $$\int_{[a;b]}f(t){\rm d}t=\int_{[a;b]}g(t){\rm d}t=\frac{1}{n}.

$$ Au début, je m'inquiétais de l'absence de l'hypothèse d'intégrabilité de Riemann, mais comme les fonctions sont continues, les fonctions sont intégrables de Riemann. Mais comment faire face à ce problème ?
Merci beaucoup à l'avance chers amis.

Réponses

  • Quantificateur sur $n$ ?
  • Ce que veut dire Chaurien, c'est que ce qui est écrit suggère qu'un choix unique fonctionne pour toutes les valeurs de $n$, ce qui est évidemment impossible. Il faut expliciter / être plus précis.
  • J'essaie de trouver la solution avec $n = 2$, mais je suppose que vous pouvez étendre la solution à n'importe quel entier n, n'est-ce pas?
  • Les deux interventions au-dessus essaient de te dire qu'il faut écrire l'énoncé correctement :

    Soit $n\in\mathbb{N}^*$. Prouver qu'il existe ...

    Le a et le b dépendent de $n$, alors que ton énoncé laisse entendre qu'ils sont indépendants de $n$.
  • Trouve la et on verra :-D
    Le 😄 Farceur


  • Je vois, je comprends. Correction de la déclaration à ce moment.
  • Tu as tout le samedi et dimanche pour le faire pour n=2
    Le 😄 Farceur


  • L'énoncé est faux de toute manière.... Il suffit de prendre $f=1$ et $g : x\mapsto 2x$ sur $[0,1]$ pour s'en rendre compte...
    Edit : j'avoue... J'étais allé trop vite ^^
  • BJ pour n=2 , ce n'est pas faux avec ton contre-exemple , prendre a=1/4 et b=3/4
    Le 😄 Farceur


  • Si $f(x)=1$ et $g(x)=2x$ sur $[0;1]$, alors comment montre-t-il qu'il n'y a pas de $a$ et $b$ tel que $[a;b]\subset [0;1]$ et $$\int_{[a;b]}f(t){\rm d}t=\int_{[a;b]}g(t){\rm d}t=\frac{1}{2}?$$
  • gebrane,

    Bien sûr.
  • Pour $x \in [0,1]$, soit $f(x)=2-2x$, $g(x)=1+\pi \cos \pi x$.
    J'envoie ceci sans avoir achevé le calcul, mais j'ai bien l'impression.que c'est un contre-exemple pour $n=2$.
  • Ah non, je croyais qu'il fallait aussi que $b-a= \frac12$, ça ne doit pas marcher.
  • Le $\frac 1n$ m'a fait penser au théorème de la corde universelle, mais je n'arrive pas à faire le lien.
  • Bonjour, imagine la fonction $H=f-g$ sur $[0,1]$. Elle sera une sorte de montée et descente juste pour dire qu'il existe un $x_0$ tel que $H(x_0)=0$ et $H$ passe par $x_0$ en changeant de signe.
    On peut trouver un voisinage $V_0$ de $x_0$ tel que $\int_{V_0}H=0 $ et $|\int_{V_0}f|\le \epsilon$ pour tout $\epsilon>0$.
    Continuant ainsi de $x_0$ à $0$ (gauche), on récupère des voisinages $V_i$ tel que
    $\int_{V_i}H=0 $ mais $\int_{V}f=1$ si $V=[0,1]$ par le théorème des valeurs intermédiaires on aura le résultat.

    Sauf erreur là.

    Cordialement.
  • Bonjour,

    Je n'ai pas eu l'idée. Avant d'utiliser le théorème des valeurs intermédiaires, je ne vois pas comment à partir de là on obtient qu'à un moment donné la somme des intégrales soit $1$.
  • Bonsoir, c'était un schéma de preuve, il faut prouver les détails, continuité etc.

    Pour ta question... laquelle ?
  • La preuve que je crois juste. Edit (Elle est incomplète)


    Soit $f$ et $g$ deux fonctions continues sur $[0;1]$ avec $\int_{[0;1]}f=\int_{[0;1]}g=1$.
    Soit $0<\epsilon<1/2$ ( Edit )
    Prouver l'existence d'un intervalle $V$ dans $[0;1]$ tel que $$\int_Vf=\int_V g=\epsilon.

    $$ On sait que $\int_{[0;1]}(f-g)=\int_{[0;1]}H =0$.
    1) Il existe un $x_0$ tel que $H(x_0)=0$ et un voisinage $V_0=[x_0-a;x_0+a]$ tel que $H(x)$ sur $[x_0-a;x_0]$ a le signe contraire que $H(x)$ sur $[x_0;x_0+a]$. (Le choix de $x_0$ est specifique).
    2) Pour tout $x_0>v>0$ soit $V_v$ l'intervalle $[x_0-v;x_0+u] \subset [0;1]$ tel que $\int_{V_v}H=0$ ici clairement $\int_{V_v}f=\int_{V_v}g$.
    3) Pour tout $\eta$ petit on peut choisir $V_{v_{\eta}}$ tel que aussi $|\int_{V_{v_{\eta}}}f|<\eta$.
    4) $\int_{[0;1]}f=1.$
    5) la fonction $v\to \int_{V_v} f$ est continue.
    Elle est à valeurs dans $[\eta;1]$. Par le théorème des valeurs intermédiaires elle passe par $\epsilon$.

    Cordialement.
  • Je m'aperçois que $\epsilon\le 1/2$ pour que ça marche. Puis le choix de $x_0$ dans le cas de plusieurs racines est truqué ou impossible. (C'est tout )

    C'est une question olympiade?

    Cordialement
  • Je me place dans le cas plus simple où l'on a de plus $f>0$ et $g>0$.

    $F(x)=\int_0^xf(t)dt$ et $G(x)=\int_0^xg(t)dt$ sont continues et strictement croissantes.

    Soit $x_k=F^{-1}(k/n)$ et $y_k=G^{-1}(k/n)$ pour $0\leq k\leq n$.

    On définit $\varphi(x)=F^{-1}(F(x)+1/n)$ pour $x\leq x_{n-1}$ et $\psi(x)=G^{-1}(G(x)+1/n)$ pour $x\leq y_{n-1}$.

    $\varphi$ et $\psi$ sont continues et croissantes. De plus, $\varphi(x_k)=x_{k+1}$ et $\psi(y_k)=y_{k+1}$.

    Soit $h(x)=\psi(x)-\varphi(x)$. S'il existe $x$ tel que $\psi(x)=\varphi(x)$ on prend $a=x$ et $b=\psi(x)$.

    Sinon, on a par exemple $h(x)>0$ pour tout $x$. On en déduit : $x_1<y_1$ puis par récurrence :

    $x_k<y_k$ entraine $x_{k+1}=\varphi(x_k)<\psi(x_k)<\psi(y_k)=y_{k+1}$. On en déduit $1=x_n<y_n=1$ : contradiction.
Connectez-vous ou Inscrivez-vous pour répondre.